Quanten.de Diskussionsforum

Quanten.de Diskussionsforum (http://www.quanten.de/forum/index.php5)
-   Quantenmechanik, Relativitätstheorie und der ganze Rest. (http://www.quanten.de/forum/forumdisplay.php5?f=3)
-   -   Winkelanteil eines Orbitals (http://www.quanten.de/forum/showthread.php5?t=2135)

baggi 14.01.12 16:10

Winkelanteil eines Orbitals
 
Hallo alle miteinander,
ich mussjetz einfach mal nachfragen, weil ich bei meiner Problematik nicht vorankomme. Und zwar habe ich folgende Aufgabenstellung:
"Skizzieren Sie den Winkelanteil eines Orbitals, dessen Wellenanteil mit
Ψ(θ,φ)=(5/8*π)^0,5 *cosθ *sinφ = (5/8*π)^0,5 * x/r gegeben ist!"

In emeinen Aufzeichnungen steht, dass der Winkelanteile nochmals aufgeteilt (separiert) sind: Ψ(θ,φ) = Θ(θ) *Η (φ)

mit: Η(φ) = (2π)^0,5 * exp (imφ)
Θ(θ) = N * "legendre-Polynom" kurz LP

aber durch das LP entstehen nur Faktoren mit dem Winkel θ.
Wie komme ich also von exp (imφ) auf das sinφ ?

Danke schonmal für die Hilfe^^

Hawkwind 14.01.12 17:32

AW: Winkelanteil eines Orbitals
 
Zitat:

Zitat von baggi (Beitrag 66016)
Hallo alle miteinander,
ich mussjetz einfach mal nachfragen, weil ich bei meiner Problematik nicht vorankomme. Und zwar habe ich folgende Aufgabenstellung:
"Skizzieren Sie den Winkelanteil eines Orbitals, dessen Wellenanteil mit
Ψ(θ,φ)=(5/8*π)^0,5 *cosθ *sinφ = (5/8*π)^0,5 * x/r gegeben ist!"

In emeinen Aufzeichnungen steht, dass der Winkelanteile nochmals aufgeteilt (separiert) sind: Ψ(θ,φ) = Θ(θ) *Η (φ)

mit: Η(φ) = (2π)^0,5 * exp (imφ)
Θ(θ) = N * "legendre-Polynom" kurz LP

aber durch das LP entstehen nur Faktoren mit dem Winkel θ.
Wie komme ich also von exp (imφ) auf das sinφ ?

Danke schonmal für die Hilfe^^

Da assoziiere ich die Identität des Hr. Euler:

http://upload.wikimedia.org/wikipedi...3d5b71d880.png

Marco Polo 14.01.12 17:38

AW: Winkelanteil eines Orbitals
 
sin(x)=(e^ix-e^-ix)/2i

Gruss, MP

Bauhof 14.01.12 17:46

AW: Winkelanteil eines Orbitals
 
Zitat:

Zitat von baggi (Beitrag 66016)
Wie komme ich also von exp (imφ) auf das sinφ ?

Hallo baggi,

was bedeutet m?

Vielleicht hilft das weiter:

sin(x) = [exp(ix) ─ exp(─ix)] / (2i)

M.f.G. Eugen Bauhof

Marco Polo 14.01.12 17:56

AW: Winkelanteil eines Orbitals
 
Hallo Eugen,

Zitat:

Zitat von Bauhof (Beitrag 66023)
was bedeutet m?

das "im" steht für Imaginärteil.

Gruss. MP

baggi 14.01.12 18:09

AW: Winkelanteil eines Orbitals
 
Hallo, also danke schonmal für die Einfälle. Aber mit den angegebenen Lösungsansätzen habe ich das schon probiert. Damit komme ich nicht wirklich voran. Habt ihr noch andere Einfälle?

m ist die Magnetquantenzahl. Man betrachtet ja hier ein Orbital mit bestimmten Quantenzahlen die mit n, l, m und s gekennzeichnet sind.

Die Thetafunktion lautet wie folgt (ich weiß ehrlich gesagt nicht, wie man hier die Formeln korrekt darstellen kann):

Θ(θ) = sqrt {(2l+1)/2 * (l - |m|!)/(l+|m|!)} P "links oben steht |m|" "links unten steht l" (cos θ) also:
Θ(θ) = Normierungsfaktor* Legendre-Polynom

für m = 0 würde ja der Exponentailanteil aber auch der phi Anteil wegfallen.
mit dem Legendre-Polynom erhalte ich nur trigonometrische Funktionen von theta.

Hawkwind 14.01.12 18:09

AW: Winkelanteil eines Orbitals
 
Zitat:

Zitat von Marco Polo (Beitrag 66024)
Hallo Eugen,



das "im" steht für Imaginärteil.

Gruss. MP

Endlich mal einen aus Herne-West erwischt! :)
Da irrst du, MP:

i = imaginäre Einheit
m = "magnetische Quantenzahl"

Der beschriebene Winkelanteil sieht nach Kugelflächenfunktion aus ("spherical harmonics"). So was kriegt man, wenn man die Schrödinger-Gleichung fürs H-Atom löst (wenn ich mich recht entsinne)

Hawkwind 14.01.12 18:11

AW: Winkelanteil eines Orbitals
 
Zitat:

Zitat von Marco Polo (Beitrag 66022)
sin(x)=(e^ix-e^-ix)/2i

Gruss, MP

Da hast du ausnahmsweise mal recht.

Marco Polo 14.01.12 18:14

AW: Winkelanteil eines Orbitals
 
Zitat:

Zitat von Hawkwind (Beitrag 66027)
Endlich mal einen aus Herne-West erwischt! :)
Da irrst du, MP:

i = imaginäre Einheit
m = "magnetische Quantenzahl"

Der beschriebene Winkelanteil sieht nach Kugelflächenfunktion aus ("spherical harmonics"). So was kriegt man, wenn man die Schrödinger-Gleichung fürs H-Atom löst (wenn ich mich recht entsinne)

Huch. Hast Recht. Na sowas...:rolleyes:

Gruss, nach Lüdenscheid-Nord

baggi 14.01.12 18:17

AW: Winkelanteil eines Orbitals
 
Zitat:

Zitat von Hawkwind (Beitrag 66027)
Der beschriebene Winkelanteil sieht nach Kugelflächenfunktion aus ("spherical harmonics"). So was kriegt man, wenn man die Schrödinger-Gleichung fürs H-Atom löst (wenn ich mich recht entsinne)

Ja die Seite habe ich auch schon angeschaut und da fand ich das hier ganz gut:

cosφsinθ = "Real" (exp (iφ) sin θ)
aber da weiß ich nicht wie ich das weiter rechnerisch bzw darstellerisch verarbeiten kann.

BTW: ich habe die Wellenfunktionen bezüglich φ und θ (also bezüglich der Wellenanteile) für l bis 2 und jeweils m = -l ..0..+l schon da aber keines davon entspricht dem, was in der Aufgabe verlangt ist. Und das sind auch nicht die selben wie auf der Wikipedia-Seite. Darf man an der Unfehlbarkeit des Professors zweifeln?^^

...Ich hoffe ich mache es nicht zu kompliziert


Alle Zeitangaben in WEZ +1. Es ist jetzt 13:36 Uhr.

Powered by vBulletin® Version 3.8.8 (Deutsch)
Copyright ©2000 - 2024, vBulletin Solutions, Inc.
ScienceUp - Dr. Günter Sturm